Prove equation in Green's function.

Click For Summary
The discussion focuses on proving an equation involving Green's function in a specified region with boundary conditions. The key equation to demonstrate is that the limit of the integral of a continuous function multiplied by the normal derivative of a harmonic function approaches zero as the radius approaches zero. The hint provided emphasizes that both the continuous function and the normal derivative are bounded near the point of interest. Participants clarify that the integration is around a small circle rather than over the boundary, and they discuss the implications of harmonicity on the boundedness of the normal derivative. The conversation concludes with a consensus that the limit can be shown to equal zero as the radius diminishes.
yungman
Messages
5,741
Reaction score
295

Homework Statement



Green's function G(x_0,y_0,x,y) =v(x_0,y_0,x,y) + h(x_0,y_0,x,y) in a region \Omega \hbox { with boundary } \Gamma. Also v(x_0,y_0,x,y) = -h(x_0,y_0,x,y) on boundary \Gamma and both v(x_0,y_0,x,y) \hbox { and }h(x_0,y_0,x,y) are harmonic function in \Omega

v=\frac{1}{2}ln[(x-x_0)^2 + (y-y_0)^2]




Let u be continuous and h is harmonic on an open disk around (x_0,y_0) in \Omega. Show that

_r\stackrel{lim}{\rightarrow}_0 \int_{C_r} u\frac{\partial h}{\partial n} ds = 0

Hint from the book: Both |u| and |\frac{\partial h}{\partial n}| are bounded near (x_0,y_0), say by M. If I_r denotes the integral in question, then |I_r| \leq 2\pi M r \rightarrow 0 \hbox { as } r\rightarrow 0

Homework Equations



Green's 1st identity:

\int _{\Omega} ( u\nabla^2 h + \nabla u \cdot \nabla h ) dx dy = \int_{\Gamma} u \frac{\partial h}{\partial n} ds




The Attempt at a Solution



h=-v \hbox { on } \Gamma \Rightarrow\; \int _{\Omega} ( u\nabla^2 h + \nabla u \cdot \nabla h ) dx dy = -\int_{\Gamma} u \frac{\partial v}{\partial n} ds = -\int_{\Gamma} u \frac{1}{r} ds = -\int^{2\pi}_{0} u d\theta

h is harmonic in \Omega\;\Rightarrow \nabla^2 h = 0

\Rightarrow\; \int _{\Omega} \nabla u \cdot \nabla h \; dx dy = -\int^{2\pi}_{0} u d\theta
I really don't know how to continue, please help.
 
Last edited:
Physics news on Phys.org
h = -v on Γ. You're not integrating over Γ; you're integrating around a small circle Cr around (x0, y0). You haven't said what h is, but by the hint, presumably ∂h/∂n is bounded on some disk centered at (x0, y0). This is certainly not true if h = -v on this disk, as ∂v/∂n = 1/r is not bounded. If h has no singularity at (x0, y0), then certainly ∂h/∂n is bounded on such a disk, as h has a continuous derivative (since by assumption h is harmonic).

Apply the hint.
 
adriank said:
h = -v on Γ. You're not integrating over Γ; you're integrating around a small circle Cr around (x0, y0). You haven't said what h is, but by the hint, presumably ∂h/∂n is bounded on some disk centered at (x0, y0). This is certainly not true if h = -v on this disk, as ∂v/∂n = 1/r is not bounded. If h has no singularity at (x0, y0), then certainly ∂h/∂n is bounded on such a disk, as h has a continuous derivative (since by assumption h is harmonic).

Apply the hint.

Thanks for you help, let me try this:

Let u be continuous and h is harmonic on an open disk around (x_0,y_0) in \Omega. Show that

_r\stackrel{lim}{\rightarrow}_0 \int_{C_r} u\frac{\partial h}{\partial n} ds = 0

Hint from the book: Both |u| and |\frac{\partial h}{\partial n}| are bounded near (x_0,y_0), say by M. If I_r denotes the integral in question, then |I_r| \leq 2\pi M r \rightarrow 0 \hbox { as } r\rightarrow 0

So |u| \hbox { and } |\frac{\partial h}{\partial n}|\; \leq\; M. As (x,y)\rightarrow\;(x_0,y_0)

_r\stackrel{lim}{\rightarrow}_0 \int_{C_r} u\frac{\partial h}{\partial n} ds = \;^+_- M^2\; \int^{2\pi}_{0} r\;d\theta \;=\; _r\stackrel{lim}{\rightarrow}_0 \; (2\pi M^2r) \;=\; 0

I have M^2 \hbox { because } u\frac{\partial h}{\partial n} \leq M^2
 
Yes. Now you have to prove that both |u| and |∂h/∂n| are bounded near that point.
 
adriank said:
Yes. Now you have to prove that both |u| and |∂h/∂n| are bounded near that point.

Thanks for your time. But I got M^2 instead of M. It does not matter because r goes to zero.
 
Question: A clock's minute hand has length 4 and its hour hand has length 3. What is the distance between the tips at the moment when it is increasing most rapidly?(Putnam Exam Question) Answer: Making assumption that both the hands moves at constant angular velocities, the answer is ## \sqrt{7} .## But don't you think this assumption is somewhat doubtful and wrong?

Similar threads

  • · Replies 6 ·
Replies
6
Views
2K
  • · Replies 10 ·
Replies
10
Views
5K
  • · Replies 2 ·
Replies
2
Views
1K
  • · Replies 1 ·
Replies
1
Views
1K
  • · Replies 0 ·
Replies
0
Views
374
  • · Replies 3 ·
Replies
3
Views
2K
  • · Replies 2 ·
Replies
2
Views
2K
  • · Replies 6 ·
Replies
6
Views
2K
  • · Replies 2 ·
Replies
2
Views
2K
  • · Replies 1 ·
Replies
1
Views
2K